Project Euler - 68 - algorithm

I have already read What is an "external node" of a "magic" 3-gon ring? and I have solved problems up until 90 but this n-gon thing totally baffles me as I don't understand the question at all.
So I take this ring and I understand that the external circles are 4, 5, 6 as they are outside the inner circle. Now he says there are eight solutions. And the eight solutions are without much explanation listed below. Let me take
9 4,2,3; 5,3,1; 6,1,2
9 4,3,2; 6,2,1; 5,1,3
So how do we arrive at the 2 solutions? I understand 4, 3, 2, is in straight line and 6,2,1 is in straight line and 5, 1, 3 are in a straight line and they are in clockwise so the second solution makes sense.
Questions
Why does the first solution 4,2,3; 5,3,1; 6,1,2 go anti clock wise? Should it not be 423 612 and then 531?
How do we arrive at 8 solutions. Is it just randomly picking three numbers? What exactly does it mean to solve a "N-gon"?

The first doesn't go anti-clockwise. It's what you get from the configuration
4
\
2
/ \
1---3---5
/
6
when you go clockwise, starting with the smallest number in the outer ring.
How do we arrive at 8 solutions. Is it just randomly picking three numbers? What exactly does it mean to solve a "N-gon"?
For an N-gon, you have an inner N-gon, and for each side of the N-gon one spike, like
X
|
X---X---X
| |
X---X---X
|
X
so that the spike together with the side of the inner N-gon connects a group of three places. A "solution" of the N-gon is a configuration where you placed the numbers from 1 to 2*N so that each of the N groups sums to the same value.
The places at the end of the spikes appear in only one group each, the places on the vertices of the inner N-gon in two. So the sum of the sums of all groups is
N
∑ k + ∑{ numbers on vertices }
k=1
The sum of the numbers on the vertices of the inner N-gon is at least 1 + 2 + ... + N = N*(N+1)/2 and at most (N+1) + (N+2) + ... + 2*N = N² + N*(N+1)/2 = N*(3*N+1)/2.
Hence the sum of the sums of all groups is between
N*(2*N+1) + N*(N+1)/2 = N*(5*N+3)/2
and
N*(2*N+1) + N*(3*N+1)/2 = N*(7*N+3)/2
inclusive, and the sum per group must be between
(5*N+3)/2
and
(7*N+3)/2
again inclusive.
For the triangle - N = 3 - the bounds are (5*3+3)/2 = 9 and (7*3+3)/2 = 12. For a square - N = 4 - the bounds are (5*4+3)/2 = 11.5 and (7*4+3)/2 = 15.5 - since the sum must be an integer, the possible sums are 12, 13, 14, 15.
Going back to the triangle, if the sum of each group is 9, the sum of the sums is 27, and the sum of the numbers on the vertices must be 27 - (1+2+3+4+5+6) = 27 - 21 = 6 = 1+2+3, so the numbers on the vertices are 1, 2 and 3.
For the sum to be 9, the value at the end of the spike for the side connecting 1 and 2 must be 6, for the side connecting 1 and 3, the spike value must be 5, and 4 for the side connecting 2 and 3.
If you start with the smallest value on the spikes - 4 - you know you have to place 2 and 3 on the vertices of the side that spike protrudes from. There are two ways to arrange the two numbers there, leading to the two solutions for sum 9.
If the sum of each group is 10, the sum of the sums is 30, and the sum of the numbers on the vertices must be 9. To represent 9 as the sum of three distinct numbers from 1 to 6, you have the possibilities
1 + 2 + 6
1 + 3 + 5
2 + 3 + 4
For the first group, you have one side connecting 1 and 2, so you'd need a 7 on the end of the spike to make 10 - no solution.
For the third group, the minimal sum of two of the numbers is 5, but 5+6 = 11 > 10, so there's no place for the 6 - no solution.
For the second group, the sums of the sides are
1 + 3 = 4 -- 6 on the spike
1 + 5 = 6 -- 4 on the spike
3 + 5 = 8 -- 2 on the spike
and you have two ways to arrange 3 and 5, so that the group is either 2-3-5 or 2-5-3, the rest follows again.
The solutions for the sums 11 and 12 can be obtained similarly, or by replacing k with 7-k in the solutions for the sums 9 resp. 10.
To solve the problem, you must now find out
what it means to obtain a 16-digit string or a 17-digit string
which sum for the groups gives rise to the largest value when the numbers are concatenated in the prescribed way.
(And use pencil and paper for the fastest solution.)

Related

Random triplet number generator

I would like to create code for a random number generator for predetermined sets of triplets (200 sets in total to randomize). I would like the sets of triplets to form a set of six numbers and the set of triplets to remain unique.
example triplets A = [1 2 3; 4 5 6; 7 8 9, 10 11 12, 13 14 15]; etc
I would like resulting triplet to retain their original sequence
1 2 3 + 4 5 6, 1 2 3 + 7 8 9, 1 2 3 + 10 11 12, 1 2 3 + 13 14 15
I am not a coder, so any help would be appreciated
You want to pick three triplets, keeping them in order. So your first triplet cannot be too close to the end -- there have to be at least two more triplets after it. Similarly, the second triplet you pick needs at least one unpicked triplet after it.
I assume that you have your triplets in an array or similar, numbered 0 to 199.
Pick a random number A in the range 0 to 197. That is the index of your first triplet.
Pick a second random number B in the range (A + 1) to 198. That is the index of your second triplet.
Pick a third random number C in the range (B + 1) to 199. That is the index of your third triplet.
The range of random numbers you pick from is affected by the numbers you have previously picked and the number of picks remaining.

Array size in Cycle leader iteration Algorithm [duplicate]

The cycle leader iteration algorithm is an algorithm for shuffling an array by moving all even-numbered entries to the front and all odd-numbered entries to the back while preserving their relative order. For example, given this input:
a 1 b 2 c 3 d 4 e 5
the output would be
a b c d e 1 2 3 4 5
This algorithm runs in O(n) time and uses only O(1) space.
One unusual detail of the algorithm is that it works by splitting the array up into blocks of size 3k+1. Apparently this is critical for the algorithm to work correctly, but I have no idea why this is.
Why is the choice of 3k + 1 necessary in the algorithm?
Thanks!
This is going to be a long answer. The answer to your question isn't simple and requires some number theory to fully answer. I've spent about half a day working through the algorithm and I now have a good answer, but I'm not sure I can describe it succinctly.
The short version:
Breaking the input into blocks of size 3k + 1 essentially breaks the input apart into blocks of size 3k - 1 surrounded by two elements that do not end up moving.
The remaining 3k - 1 elements in the block move according to an interesting pattern: each element moves to the position given by dividing the index by two modulo 3k.
This particular motion pattern is connected to a concept from number theory and group theory called primitive roots.
Because the number two is a primitive root modulo 3k, beginning with the numbers 1, 3, 9, 27, etc. and running the pattern is guaranteed to cycle through all the elements of the array exactly once and put them into the proper place.
This pattern is highly dependent on the fact that 2 is a primitive root of 3k for any k ≥ 1. Changing the size of the array to another value will almost certainly break this because the wrong property is preserved.
The Long Version
To present this answer, I'm going to proceed in steps. First, I'm going to introduce cycle decompositions as a motivation for an algorithm that will efficiently shuffle the elements around in the right order, subject to an important caveat. Next, I'm going to point out an interesting property of how the elements happen to move around in the array when you apply this permutation. Then, I'll connect this to a number-theoretic concept called primitive roots to explain the challenges involved in implementing this algorithm correctly. Finally, I'll explain why this leads to the choice of 3k + 1 as the block size.
Cycle Decompositions
Let's suppose that you have an array A and a permutation of the elements of that array. Following the standard mathematical notation, we'll denote the permutation of that array as σ(A). We can line the initial array A up on top of the permuted array σ(A) to get a sense for where every element ended up. For example, here's an array and one of its permutations:
A 0 1 2 3 4
σ(A) 2 3 0 4 1
One way that we can describe a permutation is just to list off the new elements inside that permutation. However, from an algorithmic perspective, it's often more helpful to represent the permutation as a cycle decomposition, a way of writing out a permutation by showing how to form that permutation by beginning with the initial array and then cyclically permuting some of its elements.
Take a look at the above permutation. First, look at where the 0 ended up. In σ(A), the element 0 ended up taking the place of where the element 2 used to be. In turn, the element 2 ended up taking the place of where the element 0 used to be. We denote this by writing (0 2), indicating that 0 should go where 2 used to be, and 2 should go were 0 used to be.
Now, look at the element 1. The element 1 ended up where 4 used to be. The number 4 then ended up where 3 used to be, and the element 3 ended up where 1 used to be. We denote this by writing (1 4 3), that 1 should go where 4 used to be, that 4 should go where 3 used to be, and that 3 should go where 1 used to be.
Combining these together, we can represent the overall permutation of the above elements as (0 2)(1 4 3) - we should swap 0 and 2, then cyclically permute 1, 4, and 3. If we do that starting with the initial array, we'll end up at the permuted array that we want.
Cycle decompositions are extremely useful for permuting arrays in place because it's possible to permute any individual cycle in O(C) time and O(1) auxiliary space, where C is the number of elements in the cycle. For example, suppose that you have a cycle (1 6 8 4 2). You can permute the elements in the cycle with code like this:
int[] cycle = {1, 6, 8, 4, 2};
int temp = array[cycle[0]];
for (int i = 1; i < cycle.length; i++) {
swap(temp, array[cycle[i]]);
}
array[cycle[0]] = temp;
This works by just swapping everything around until everything comes to rest. Aside from the space usage required to store the cycle itself, it only needs O(1) auxiliary storage space.
In general, if you want to design an algorithm that applies a particular permutation to an array of elements, you can usually do so by using cycle decompositions. The general algorithm is the following:
for (each cycle in the cycle decomposition algorithm) {
apply the above algorithm to cycle those elements;
}
The overall time and space complexity for this algorithm depends on the following:
How quickly can we determine the cycle decomposition we want?
How efficiently can we store that cycle decomposition in memory?
To get an O(n)-time, O(1)-space algorithm for the problem at hand, we're going to show that there's a way to determine the cycle decomposition in O(1) time and space. Since everything will get moved exactly once, the overall runtime will be O(n) and the overall space complexity will be O(1). It's not easy to get there, as you'll see, but then again, it's not awful either.
The Permutation Structure
The overarching goal of this problem is to take an array of 2n elements and shuffle it so that even-positioned elements end up at the front of the array and odd-positioned elements end up at the end of the array. Let's suppose for now that we have 14 elements, like this:
0 1 2 3 4 5 6 7 8 9 10 11 12 13
We want to shuffle the elements so that they come out like this:
0 2 4 6 8 10 12 1 3 5 7 9 11 13
There are a couple of useful observations we can have about the way that this permutation arises. First, notice that the first element does not move in this permutation, because even-indexed elements are supposed to show up in the front of the array and it's the first even-indexed element. Next, notice that the last element does not move in this permutation, because odd-indexed elements are supposed to end up at the back of the array and it's the last odd-indexed element.
These two observations, put together, means that if we want to permute the elements of the array in the desired fashion, we actually only need to permute the subarray consisting of the overall array with the first and last elements dropped off. Therefore, going forward, we are purely going to focus on the problem of permuting the middle elements. If we can solve that problem, then we've solved the overall problem.
Now, let's look at just the middle elements of the array. From our above example, that means that we're going to start with an array like this one:
Element 1 2 3 4 5 6 7 8 9 10 11 12
Index 1 2 3 4 5 6 7 8 9 10 11 12
We want to get the array to look like this:
Element 2 4 6 8 10 12 1 3 5 7 9 11
Index 1 2 3 4 5 6 7 8 9 10 11 12
Because this array was formed by taking a 0-indexed array and chopping off the very first and very last element, we can treat this as a one-indexed array. That's going to be critically important going forward, so be sure to keep that in mind.
So how exactly can we go about generating this permutation? Well, for starters, it doesn't hurt to take a look at each element and to try to figure out where it began and where it ended up. If we do so, we can write things out like this:
The element at position 1 ended up at position 7.
The element at position 2 ended up at position 1.
The element at position 3 ended up at position 8.
The element at position 4 ended up at position 2.
The element at position 5 ended up at position 9.
The element at position 6 ended up at position 3.
The element at position 7 ended up at position 10.
The element at position 8 ended up at position 4.
The element at position 9 ended up at position 11.
The element at position 10 ended up at position 5.
The element at position 11 ended up at position 12.
The element at position 12 ended up at position 6.
If you look at this list, you can spot a few patterns. First, notice that the final index of all the even-numbered elements is always half the position of that element. For example, the element at position 4 ended up at position 2, the element at position 12 ended up at position 6, etc. This makes sense - we pushed all the even elements to the front of the array, so half of the elements that came before them will have been displaced and moved out of the way.
Now, what about the odd-numbered elements? Well, there are 12 total elements. Each odd-numbered element gets pushed to the second half, so an odd-numbered element at position 2k+1 will get pushed to at least position 7. Its position within the second half is given by the value of k. Therefore, the elements at an odd position 2k+1 gets mapped to position 7 + k.
We can take a minute to generalize this idea. Suppose that the array we're permuting has length 2n. An element at position 2x will be mapped to position x (again, even numbers get halfed), and an element at position 2x+1 will be mapped to position n + 1 + x. Restating this:
The final position of an element at position p is determined as follows:
If p = 2x for some integer x, then 2x ↦ x
If p = 2x+1 for some integer x, then 2x+1 ↦ n + 1 + x
And now we're going to do something that's entirely crazy and unexpected. Right now, we have a piecewise rule for determining where each element ends up: we either divide by two, or we do something weird involving n + 1. However, from a number-theoretic perspective, there is a single, unified rule explaining where all elements are supposed to end up.
The insight we need is that in both cases, it seems like, in some way, we're dividing the index by two. For the even case, the new index really is formed by just dividing by two. For the odd case, the new index kinda looks like it's formed by dividing by two (notice that 2x+1 went to x + (n + 1)), but there's an extra term in there. In a number-theoretic sense, though, both of these really correspond to division by two. Here's why.
Rather than taking the source index and dividing by two to get the destination index, what if we take the destination index and multiply by two? If we do that, an interesting pattern emerges.
Suppose our original number was 2x. The destination is then x, and if we double the destination index to get back 2x, we end up with the source index.
Now suppose that our original number was 2x+1. The destination is then n + 1 + x. Now, what happens if we double the destination index? If we do that, we get back 2n + 2 + 2x. If we rearrange this, we can alternatively rewrite this as (2x+1) + (2n+1). In other words, we've gotten back the original index, plus an extra (2n+1) term.
Now for the kicker: what if all of our arithmetic is done modulo 2n + 1? In that case, if our original number was 2x + 1, then twice the destination index is (2x+1) + (2n+1) = 2x + 1 (modulo 2n+1). In other words, the destination index really is half of the source index, just done modulo 2n+1!
This leads us to a very, very interesting insight: the ultimate destination of each of the elements in a 2n-element array is given by dividing that number by two, modulo 2n+1. This means that there really is a nice, unified rule for determining where everything goes. We just need to be able to divide by two modulo 2n+1. It just happens to work out that in the even case, this is normal integer division, and in the odd case, it works out to taking the form n + 1 + x.
Consequently, we can reframe our problem in the following way: given a 1-indexed array of 2n elements, how do we permute the elements so that each element that was originally at index x ends up at position x/2 mod (2n+1)?
Cycle Decompositions Revisited
At this point, we've made quite a lot of progress. Given any element, we know where that element should end up. If we can figure out a nice way to get a cycle decomposition of the overall permutation, we're done.
This is, unfortunately, where things get complicated. Suppose, for example, that our array has 10 elements. In that case, we want to transform the array like this:
Initial: 1 2 3 4 5 6 7 8 9 10
Final: 2 4 6 8 10 1 3 5 7 9
The cycle decomposition of this permutation is (1 6 3 7 9 10 5 8 4 2). If our array has 12 elements, we want to transform it like this:
Initial: 1 2 3 4 5 6 7 8 9 10 11 12
Final: 2 4 6 8 10 12 1 3 5 7 9 11
This has cycle decomposition (1 7 10 5 9 11 12 6 3 8 4 2 1). If our array has 14 elements, we want to transform it like this:
Initial: 1 2 3 4 5 6 7 8 9 10 11 12 13 14
Final: 2 4 6 8 10 12 14 1 3 5 7 9 11 13
This has cycle decomposition (1 8 4 2)(3 9 12 6)(5 10)(7 11 13 14). If our array has 16 elements, we want to transform it like this:
Initial: 1 2 3 4 5 6 7 8 9 10 11 12 13 14 15 16
Final: 2 4 6 8 10 12 14 16 1 3 5 7 9 11 13 15
This has cycle decomposition (1 9 13 15 16 8 4 2)(3 10 5 11 14 7 12 6).
The problem here is that these cycles don't seem to follow any predictable patterns. This is a real problem if we're going to try to solve this problem in O(1) space and O(n) time. Even though given any individual element we can figure out what cycle contains it and we can efficiently shuffle that cycle, it's not clear how we figure out what elements belong to what cycles, how many different cycles there are, etc.
Primitive Roots
This is where number theory comes in. Remember that each element's new position is formed by dividing that number by two, modulo 2n+1. Thinking about this backwards, we can figure out which number will take the place of each number by multiplying by two modulo 2n+1. Therefore, we can think of this problem by finding the cycle decomposition in reverse: we pick a number, keep multiplying it by two and modding by 2n+1, and repeat until we're done with the cycle.
This gives rise to a well-studied problem. Suppose that we start with the number k and think about the sequence k, 2k, 22k, 23k, 24k, etc., all done modulo 2n+1. Doing this gives different patterns depending on what odd number 2n+1 you're modding by. This explains why the above cycle patterns seem somewhat arbitrary.
I have no idea how anyone figured this out, but it turns out that there's a beautiful result from number theory that talks about what happens if you take this pattern mod 3k for some number k:
Theorem: Consider the sequence 3s, 3s·2, 3s·22, 3s·23, 3s·24, etc. all modulo 3k for some k ≥ s. This sequence cycles through through every number between 1 and 3k, inclusive, that is divisible by 3s but not divisible by 3s+1.
We can try this out on a few examples. Let's work modulo 27 = 32. The theorem says that if we look at 3, 3 · 2, 3 · 4, etc. all modulo 27, then we should see all the numbers less than 27 that are divisible by 3 and not divisible by 9. Well, let'see what we get:
3 · 20 = 3 · 1 = 3 = 3 mod 27
3 · 21 = 3 · 2 = 6 = 6 mod 27
3 · 22 = 3 · 4 = 12 = 12 mod 27
3 · 23 = 3 · 8 = 24 = 24 mod 27
3 · 24 = 3 · 16 = 48 = 21 mod 27
3 · 25 = 3 · 32 = 96 = 15 mod 27
3 · 26 = 3 · 64 = 192 = 3 mod 27
We ended up seeing 3, 6, 12, 15, 21, and 24 (though not in that order), which are indeed all the numbers less than 27 that are divisible by 3 but not divisible by 9.
We can also try this working mod 27 and considering 1, 2, 22, 23, 24 mod 27, and we should see all the numbers less than 27 that are divisible by 1 and not divisible by 3. In other words, this should give back all the numbers less than 27 that aren't divisible by 3. Let's see if that's true:
20 = 1 = 1 mod 27
21 = 2 = 2 mod 27
22 = 4 = 4 mod 27
23 = 8 = 8 mod 27
24 = 16 = 16 mod 27
25 = 32 = 5 mod 27
26 = 64 = 10 mod 27
27 = 128 = 20 mod 27
28 = 256 = 13 mod 27
29 = 512 = 26 mod 27
210 = 1024 = 25 mod 27
211 = 2048 = 23 mod 27
212 = 4096 = 19 mod 27
213 = 8192 = 11 mod 27
214 = 16384 = 22 mod 27
215 = 32768 = 17 mod 27
216 = 65536 = 7 mod 27
217 = 131072 = 14 mod 27
218 = 262144 = 1 mod 27
Sorting these, we got back the numbers 1, 2, 4, 5, 7, 8, 10, 11, 13, 14, 16, 17, 19, 20, 22, 23, 25, 26 (though not in that order). These are exactly the numbers between 1 and 26 that aren't multiples of three!
This theorem is crucial to the algorithm for the following reason: if 2n+1 = 3k for some number k, then if we process the cycle containing 1, it will properly shuffle all numbers that aren't multiples of three. If we then start the cycle at 3, it will properly shuffle all numbers that are divisible by 3 but not by 9. If we then start the cycle at 9, it will properly shuffle all numbers that are divisible by 9 but not by 27. More generally, if we use the cycle shuffle algorithm on the numbers 1, 3, 9, 27, 81, etc., then we will properly reposition all the elements in the array exactly once and will not have to worry that we missed anything.
So how does this connect to 3k + 1? Well, we need to have that 2n + 1 = 3k, so we need to have that 2n = 3k - 1. But remember - we dropped the very first and very last element of the array when we did this! Adding those back in tells us that we need blocks of size 3k + 1 for this procedure to work correctly. If the blocks are this size, then we know for certain that the cycle decomposition will consist of a cycle containing 1, a nonoverlapping cycle containing 3, a nonoverlapping cycle containing 9, etc. and that these cycles will contain all the elements of the array. Consequently, we can just start cycling 1, 3, 9, 27, etc. and be absolutely guaranteed that everything gets shuffled around correctly. That's amazing!
And why is this theorem true? It turns out that a number k for which 1, k, k2, k3, etc. mod pn that cycles through all the numbers that aren't multiples of p (assuming p is prime) is called a primitive root of the number pn. There's a theorem that says that 2 is a primitive root of 3k for all numbers k, which is why this trick works. If I have time, I'd like to come back and edit this answer to include a proof of this result, though unfortunately my number theory isn't at a level where I know how to do this.
Summary
This problem was tons of fun to work on. It involves cute tricks with dividing by two modulo an odd numbers, cycle decompositions, primitive roots, and powers of three. I'm indebted to this arXiv paper which described a similar (though quite different) algorithm and gave me a sense for the key trick behind the technique, which then let me work out the details for the algorithm you described.
Hope this helps!
Here is most of the mathematical argument missing from templatetypedef’s
answer. (The rest is comparatively boring.)
Lemma: for all integers k >= 1, we have
2^(2*3^(k-1)) = 1 + 3^k mod 3^(k+1).
Proof: by induction on k.
Base case (k = 1): we have 2^(2*3^(1-1)) = 4 = 1 + 3^1 mod 3^(1+1).
Inductive case (k >= 2): if 2^(2*3^(k-2)) = 1 + 3^(k-1) mod 3^k,
then q = (2^(2*3^(k-2)) - (1 + 3^(k-1)))/3^k.
2^(2*3^(k-1)) = (2^(2*3^(k-2)))^3
= (1 + 3^(k-1) + 3^k*q)^3
= 1 + 3*(3^(k-1)) + 3*(3^(k-1))^2 + (3^(k-1))^3
+ 3*(1+3^(k-1))^2*(3^k*q) + 3*(1+3^(k-1))*(3^k*q)^2 + (3^k*q)^3
= 1 + 3^k mod 3^(k+1).
Theorem: for all integers i >= 0 and k >= 1, we have
2^i = 1 mod 3^k if and only if i = 0 mod 2*3^(k-1).
Proof: the “if” direction follows from the Lemma. If
i = 0 mod 2*3^(k-1), then
2^i = (2^(2*3^(k-1)))^(i/(2*3^(k-1)))
= (1+3^k)^(i/(2*3^(k-1))) mod 3^(k+1)
= 1 mod 3^k.
The “only if” direction is by induction on k.
Base case (k = 1): if i != 0 mod 2, then i = 1 mod 2, and
2^i = (2^2)^((i-1)/2)*2
= 4^((i-1)/2)*2
= 2 mod 3
!= 1 mod 3.
Inductive case (k >= 2): if 2^i = 1 mod 3^k, then
2^i = 1 mod 3^(k-1), and the inductive hypothesis implies that
i = 0 mod 2*3^(k-2). Let j = i/(2*3^(k-2)). By the Lemma,
1 = 2^i mod 3^k
= (1+3^(k-1))^j mod 3^k
= 1 + j*3^(k-1) mod 3^k,
where the dropped terms are divisible by (3^(k-1))^2, so
j = 0 mod 3, and i = 0 mod 2*3^(k-1).

Strategy with regard to how to approach this algorithm?

I was asked this question in a test and I need help with regards to how I should approach the solution, not the actual answer. The question is
You have been given a 7 digit number(with each digit being distinct and 0-9). The number has this property
product of first 3 digits = product of last 3 digits = product of central 3 digits
Identify the middle digit.
Now, I can do this on paper by brute force(trial and error), the product is 72 and digits being
8,1,9,2,4,3,6
Now how do I approach the problem in a no brute force way?
Let the number is: a b c d e f g
So as per the rule(1):
axbxc = cxdxe = exfxg
more over we have(2):
axb = dxe and
cxd = fxg
This question can be solved with factorization and little bit of hit/trial.
Out of the digits from 1 to 9, 5 and 7 can rejected straight-away since these are prime numbers and would not fit in the above two equations.
The digits 1 to 9 can be factored as:
1 = 1, 2 = 2, 3 = 3, 4 = 2X2, 6 = 2X3, 8 = 2X2X2, 9 = 3X3
After factorization we are now left with total 7 - 2's, 4 - 3's and the number 1.
As for rule 2 we are left with only 4 possibilities, these 4 equations can be computed by factorization logic since we know we have overall 7 2's and 4 3's with us.
1: 1X8(2x2x2) = 2X4(2x2)
2: 1X6(3x2) = 3X2
3: 4(2x2)X3 = 6(3x2)X2
4: 9(3x3)X2 = 6(3x2)X3
Skipping 5 and 7 we are left with 7 digits.
With above equations we have 4 digits with us and are left with remaining 3 digits which can be tested through hit and trial. For example, if we consider the first case we have:
1X8 = 2X4 and are left with 3,6,9.
we have axbxc = cxdxe we can opt c with these 3 options in that case the products would be 24, 48 and 72.
24 cant be correct since for last three digits we are left with are 6,9,4(=216)
48 cant be correct since for last three digits we are left with 3,9,4(=108)
72 could be a valid option since the last three digits in that case would be 3,6,4 (=72)
This question is good to solve with Relational Programming. I think it very clearly lets the programmer see what's going on and how the problem is solved. While it may not be the most efficient way to solve problems, it can still bring desired clarity and handle problems up to a certain size. Consider this small example from Oz:
fun {FindDigits}
D1 = {Digit}
D2 = {Digit}
D3 = {Digit}
D4 = {Digit}
D5 = {Digit}
D6 = {Digit}
D7 = {Digit}
L = [D1 D2 D3] M = [D3 D4 D5] E= [D5 D6 D7] TotL in
TotL = [D1 D2 D3 D4 D5 D6 D7]
{Unique TotL} = true
{ProductList L} = {ProductList M} = {ProductList E}
TotL
end
(Now this would be possible to parameterize furthermore, but non-optimized to illustrate the point).
Here you first pick 7 digits with a function Digit/0. Then you create three lists, L, M and E consisting of the segments, as well as a total list to return (you could also return the concatenation, but I found this better for illustration).
Then comes the point, you specify relations that have to be intact. First, that the TotL is unique (distinct in your tasks wording). Then the next one, that the segment products have to be equal.
What now happens is that a search is conducted for your answers. This is a depth-first search strategy, but could also be breadth-first, and a solver is called to bring out all solutions. The search strategy is found inside the SolveAll/1 function.
{Browse {SolveAll FindDigits}}
Which in turns returns this list of answers:
[[1 8 9 2 4 3 6] [1 8 9 2 4 6 3] [3 6 4 2 9 1 8]
[3 6 4 2 9 8 1] [6 3 4 2 9 1 8] [6 3 4 2 9 8 1]
[8 1 9 2 4 3 6] [8 1 9 2 4 6 3]]
At least this way forward is not using brute force. Essentially you are searching for answers here. There might be heuristics that let you find the correct answer sooner (some mathematical magic, perhaps), or you can use genetic algorithms to search the space or other well-known strategies.
Prime factor of distinct digit (if possible)
0 = 0
1 = 1
2 = 2
3 = 3
4 = 2 x 2
5 = 5
6 = 2 x 3
7 = 7
8 = 2 x 2 x 2
9 = 3 x 3
In total:
7 2's + 4 3's + 1 5's + 1 7's
With the fact that When A=B=C, composition of prime factor of A must be same as composition of prime factor of B and that of C, 0 , 5 and 7 are excluded since they have unique prime factor that can never match with the fact.
Hence, 7 2's + 4 3's are left and we have 7 digit (1,2,3,4,6,8,9). As there are 7 digits only, the number is formed by these digits only.
Recall the fact, A, B and C must have same composition of prime factors. This implies that A, B and C have same number of 2's and 3's in their composition. So, we should try to achieve (in total for A and B and C):
9 OR 12 2's AND
6 3's
(Must be product of 3, lower bound is total number of prime factor of all digits, upper bound is lower bound * 2)
Consider point 2 (as it has one possibility), A has 2 3's and same for B and C. To have more number of prime factor in total, we need to put digit in connection digit between two product (third or fifth digit). Extract digits with prime factor 3 into two groups {3,6} and {9} and put digit into connection digit. The only possible way is to put 9 in connection digit and 3,6 on unconnected product. That mean xx9xx36 or 36xx9xx (order of 3,6 is not important)
With this result, we get 9 x middle x connection digit = connection digit x 3 x 6. Thus, middle = (3 x 6) / 9 = 2
My answer actually extends #Ansh's answer.
Let abcdefg be the digits of the number. Then
ab=de
cd=fg
From these relations we can exclude 0, 5 and 7 because there are no other multipliers of these numbers between 0 and 9. So we are left with seven numbers and each number is included once in each answer. We are going to examine how we can pair the numbers (ab, de, cd, fg).
What happens with 9? It can't be combined with 3 or 6 since then their product will have three times the factor 3 and we have at total 4 factors of 3. Similarly, 3 and 6 must be combined at least one time together in response to the two factors of 9. This gives a product of 18 and so 9 must be combined at least once with 2.
Now if 9x2 is in a corner then 3x6 must be in the middle. Meaning in the other corner there must be another multiplier of 3. So 9 and 2 are in the middle.
Let's suppose ab=3x6 (The other case is symmetric). Then d must be 9 or 2. But if d is 9 then f or g must be multiplier of 3. So d is 2 and e is 9. We can stop here and answer the middle digit is
2
Now we have 2c = fg and the remaining choices are 1, 4, 8. We see that the only solutions are c = 4, f = 1, g = 8 and c = 4, f = 8, g = 1.
So if is 3x6 is in the left corner we have the following solutions:
3642918, 3642981, 6342918, 6342981
If 3x6 is in the right corner we have the following solutions which are the reverse of the above:
8192463, 1892463, 8192436, 1892436
Here is how you can consider the problem:
Let's note the final solution N1 N2 N3 N4 N5 N6 N7 for the 3 numbers N1N2N3, N3N4N5 and N5N6N7
0, 5 and 7 are to exclude because they are prime and no other ciphers is a multiple of them. So if they had divided one of the 3 numbers, no other number could have divided the others.
So we get the 7 remaining ciphers : 1234689
where the product of the ciphers is 2^7*3^4
(N1*N2*N3) and (N5*N6*N7) are equals so their product is a square number. We can then remove, one of the number (N4) from the product of the previous point to find a square number (i.e. even exponents on both numbers)
N4 can't be 1, 3, 4, 6, 9.
We conclude N4 is 2 or 8
If N4 is 8 and it divides (N3*N4*N5), we can't use the remaining even numbers (2, 4, 6) to divides
both (N1*N2*N3) and (N6*N7*N8) by 8. So N4 is 2 and 8 does not belong to the second group (let's put it in N1).
Now, we have: 1st grp: 8XX, 2nd group: X2X 3rd group: XXX
Note: at this point we know that the product is 72 because it is 2^3*3^2 (the square root of 2^6*3^4) but the result is not really important. We have made the difficult part knowing the 7 numbers and the middle position.
Then, we know that we have to distribute 2^3 on (N1*N2*N3), (N3*N4*N5), (N5*N6*N7) because 2^3*2*2^3=2^7
We already gave 8 to N1, 2 to N4 and we place 6 to N6, and 4 to N5 position, resulting in each of the 3 numbers being a multiple of 8.
Now, we have: 1st grp: 8XX, 2nd group: X24 3rd group: 46X
We have the same way of thinking considering the odd number, we distribute 3^2, on each part knowing that we already have a 6 in the last group.
Last group will then get the 3. And first and second ones the 9.
Now, we have: 1st grp: 8X9, 2nd group: 924 3rd group: 463
And, then 1 at N2, which is the remaining position.
This problem is pretty easy if you look at the number 72 more carefully.
We have our number with this form abcdefg
and abc = cde = efg, with those digits 8,1,9,2,4,3,6
So, first, we can conclude that 8,1,9 must be one of the triple, because, there is no way 1 can go with other two numbers to form 72.
We can also conclude that 1 must be in the start/end of the whole number or middle of the triple.
So now we have 819defg or 918defg ...
Using some calculations with the rest of those digits, we can see that only 819defg is possible, because, we need 72/9 = 8,so only 2,4 is valid, while we cannot create 72/8 = 9 from those 2,4,3,6 digits, so -> 81924fg or 81942fg and 819 must be the triple that start or end our number.
So the rest of the job is easy, we need either 72/4 = 18 or 72/2 = 36, now, we can have our answers: 8192436 or 8192463.
7 digits: 8,1,9,2,4,3,6
say XxYxZ = 72
1) pick any two from above 7 digits. say X,Y
2) divide 72 by X and then Y.. you will get the 3rd number i.e Z.
we found XYZ set of 3-digits which gives result 72.
now repeat 1) and 2) with remaining 4 digits.
this time we found ABC which multiplies to 72.
lets say, 7th digit left out is I.
3) divide 72 by I. result R
4) divide R by one of XYZ. check if result is in ABC.
if No, repeat the step 3)
if yes, found the third pair.(assume you divided R by Y and the result is B)
YIB is the third pair.
so... solution will be.
XZYIBAC
You have your 7 numbers - instead of looking at it in groups of 3 divide up the number as such:
AB | C | D | E | FG
Get the value of AB and use it to get the value of C like so: C = ABC/AB
Next you want to do the same thing with the trailing 2 digits to find E using FG. E = EFG/FG
Now that you have C & E you can solve for D
Since CDE = ABC then D = ABC/CE
Remember your formulas - instead of looking at numbers create a formula aka an algorithm that you know will work every time.
ABC = CDE = EFG However, you have to remember that your = signs have to balance. You can see that D = ABC/CE = EFG/CE Once you know that, you can figure out what you need in order to solve the problem.
Made a quick example in a fiddle of the code:
http://jsfiddle.net/4ykxx9ve/1/
var findMidNum = function() {
var num = [8, 1, 9, 2, 4, 3, 6];
var ab = num[0] * num[1];
var fg = num[5] * num[6];
var abc = num[0] * num[1] * num[2];
var cde = num[2] * num[3] * num[4];
var efg = num[4] * num[5] * num[6];
var c = abc/ab;
var e = efg/fg;
var ce = c * e
var d = abc/ce;
console.log(d); //2
}();
You have been given a 7 digit number(with each digit being distinct and 0-9). The number has this property
product of first 3 digits = product of last 3 digits = product of central 3 digits
Identify the middle digit.
Now, I can do this on paper by brute force(trial and error), the product is 72 and digits being
8,1,9,2,4,3,6
Now how do I approach the problem in a no brute force way?
use linq and substring functions
example var item = array.Skip(3).Take(3) in such a way that you have a loop
for(f =0;f<charlen.length;f++){
var xItemSum = charlen[f].Skip(f).Take(f).Sum(f => f.Value);
}
// untested code

Networking Algorithm to Maximum a Subset of Integers

I'm currently working on a pet project that simulates a couple different types of networks. One of them requires some specific conditions that until now I've just been brute forcing. It's not scaling well, however, so I'm trying to do this efficiently but this algorithm is really stumping me! I'll try to describe the problem as general as possible.
Given a set of integers X and an integer k, find a subset Y of X that maximizes the sum of M over each value in X:
M(s) = the largest value in Y such that it is less than or equal to s.
For example, for {2, 4, 5} and k = 2 the solution is {2, 4} with value 2+4+4=10 since M(2) = 2, M(4) = 4, and M(5) = 5.
My intuition is that the solution is a dynamic programming algorithm, but I could be way off. Any help would be greatly appreciated!
Here is a dynamic program problem with a solution - I'm not sure if it's yours because I'm not sure of the details of what you have written, but it might be.
Sort the set of numbers and draw a curve with the x axis giving the offset of the number in sorted order and the y axis giving the number. There will be some area under the curve.
You have a finite number of points, usually a smaller number than there are members of the set. You can use each of these points to mark a point of the set, and so a point of the curve.
Draw a histogram under the curve. At each marked point there is a line from that point going right, so the lines are entirely under the curve. Each such line extends till it reaches the x value for the next marked point, at which point there is a line going up to the new marked point.
The challenge is then to select which points to mark to maximize the area under the horizontal lines going right from marked points. This is straightforward dynamic programming. If you can choose up to k marked points then at each point of the histogram work out the most area you can cover to the left of that point using 0, 1, 2, ..k marked points, possibly including that point. You can work out the answer for each point by referring to the answers you have already worked out for the points to its left. The answer for the rightmost point is the answer for the entire problem.
To expand this: Suppose you are working out the best solutions for maximum area ending at offset 10. For each value j of 0..k consider taking the previous best solution ending at 0, 1, 2, 3... 9 and maintaining the height at that point, without introducing a new line. The total area for this is the area up to that point plus the new area gained by whatever height they were in at that point times the distance back to that point. Also consider doing this, but using an extra marked point at that point, so the total area is the area of the best solution with j-1 points up to e.g. point 7 plus the distance back from point 10 to point 7 times the height reached at point 7. By considering these two possibilities you can work out the best solution at point 10 using 0,1,2,...k marked points.
I think these problems are related because for each point, marked or not, the area it contributes to the histogram depends on the height of the line above it, which is the height of the largest marked point no greater that the point we are considering at the moment.
To do this you need an array of kn elements giving the area covered by the best solution at each point using at most k marked points up to there. It will also be convenient to use an extra array of this size to record the decision that led to this best solution, so you can trace the answer back. This has a cost of about kn^2, because at each of n points you need to calculate k values, and look back at all the previous points as you do so. I suspect that you could reduce this to something like O(kn) by changing the definition of what you store at each point so you never have to look back further than one previous point. If you could do that, you could economize on store at the cost of time by only storing a few intermediate points and solving the problem over again on smaller sections to trace back, but you'ld need to be desperately short of store to make that worth while.
My answer is very similar than the other one:
The algorithm I suggest is to start having K=N, all numbers ordered, and keep removing numbers until you reach the desired K. The number you select to remove in each step, is the one who represent the lowest loss.
Example: Let's say you have the numbers:
3, 7, 9, 13 and 19
The problem is K=3
You start in K=5 (all numbers are selected).
3 + 7 + 9 + 13 + 19 = 51
First number to remove:
if 3 is selected:
0 + 7 + 9 + 13 + 19 = 48 (we lose 3)
if 4 is selected: (7 becomes 3)
3 + 3 + 9 + 13 + 19 = 47 (we lose 4)
if 9 is selected: (9 becomes 7)
3 + 7 + 7 + 13 + 19 = 49 (we lose 2)
if 13 is selected: we lose 13 - 9 = 4
if 19 is selected: we lose 19 - 13 = 6
Lowest loss in this case is: number 9 (loss=2).
We remove 9, and then we have K=4.
For the second number to remove, we have 4 options:
if we remove 3:
0 + 7 + 7 + 13 + 19 = # (we lose 3)
if we remove 7 all 7s will become 3s:
3 + 3 + 3 + 13 + 19 = # (we lose two 7s becoming 3 = (7-3) x 2 = 8)
if we remove 13:
3 + 7 + 7 + 7 + 19 (loss = 13 - 7 = 6)
if 19 is removed:
3 + 7 + 7 + 13 + 13 (loss = 6)
Best selection here is to remove #3
and then K=3 achieving the sum: 46
I don't know if this is optimal, you could verify by comparing vs. brute force multiple cases. But, even if this is not optimal, it can give good results.

minimum steps required to make array of integers contiguous

given a sorted array of distinct integers, what is the minimum number of steps required to make the integers contiguous? Here the condition is that: in a step , only one element can be changed and can be either increased or decreased by 1 . For example, if we have 2,4,5,6 then '2' can be made '3' thus making the elements contiguous(3,4,5,6) .Hence the minimum steps here is 1 . Similarly for the array: 2,4,5,8:
Step 1: '2' can be made '3'
Step 2: '8' can be made '7'
Step 3: '7' can be made '6'
Thus the sequence now is 3,4,5,6 and the number of steps is 3.
I tried as follows but am not sure if its correct?
//n is the number of elements in array a
int count=a[n-1]-a[0]-1;
for(i=1;i<=n-2;i++)
{
count--;
}
printf("%d\n",count);
Thanks.
The intuitive guess is that the "center" of the optimal sequence will be the arithmetic average, but this is not the case. Let's find the correct solution with some vector math:
Part 1: Assuming the first number is to be left alone (we'll deal with this assumption later), calculate the differences, so 1 12 3 14 5 16-1 2 3 4 5 6 would yield 0 -10 0 -10 0 -10.
sidenote: Notice that a "contiguous" array by your implied definition would be an increasing arithmetic sequence with difference 1. (Note that there are other reasonable interpretations of your question: some people may consider 5 4 3 2 1 to be contiguous, or 5 3 1 to be contiguous, or 1 2 3 2 3 to be contiguous. You also did not specify if negative numbers should be treated any differently.)
theorem: The contiguous numbers must lie between the minimum and maximum number. [proof left to reader]
Part 2: Now returning to our example, assuming we took the 30 steps (sum(abs(0 -10 0 -10 0 -10))=30) required to turn 1 12 3 14 5 16 into 1 2 3 4 5 6. This is one correct answer. But 0 -10 0 -10 0 -10+c is also an answer which yields an arithmetic sequence of difference 1, for any constant c. In order to minimize the number of "steps", we must pick an appropriate c. In this case, each time we increase or decrease c, we increase the number of steps by N=6 (the length of the vector). So for example if we wanted to turn our original sequence 1 12 3 14 5 16 into 3 4 5 6 7 8 (c=2), then the differences would have been 2 -8 2 -8 2 -8, and sum(abs(2 -8 2 -8 2 -8))=30.
Now this is very clear if you could picture it visually, but it's sort of hard to type out in text. First we took our difference vector. Imagine you drew it like so:
4|
3| *
2| * |
1| | | *
0+--+--+--+--+--*
-1| |
-2| *
We are free to "shift" this vector up and down by adding or subtracting 1 from everything. (This is equivalent to finding c.) We wish to find the shift which minimizes the number of | you see (the area between the curve and the x-axis). This is NOT the average (that would be minimizing the standard deviation or RMS error, not the absolute error). To find the minimizing c, let's think of this as a function and consider its derivative. If the differences are all far away from the x-axis (we're trying to make 101 112 103 114 105 116), it makes sense to just not add this extra stuff, so we shift the function down towards the x-axis. Each time we decrease c, we improve the solution by 6. Now suppose that one of the *s passes the x axis. Each time we decrease c, we improve the solution by 5-1=4 (we save 5 steps of work, but have to do 1 extra step of work for the * below the x-axis). Eventually when HALF the *s are past the x-axis, we can NO LONGER IMPROVE THE SOLUTION (derivative: 3-3=0). (In fact soon we begin to make the solution worse, and can never make it better again. Not only have we found the minimum of this function, but we can see it is a global minimum.)
Thus the solution is as follows: Pretend the first number is in place. Calculate the vector of differences. Minimize the sum of the absolute value of this vector; do this by finding the median OF THE DIFFERENCES and subtracting that off from the differences to obtain an improved differences-vector. The sum of the absolute value of the "improved" vector is your answer. This is O(N) The solutions of equal optimality will (as per the above) always be "adjacent". A unique solution exists only if there are an odd number of numbers; otherwise if there are an even number of numbers, AND the median-of-differences is not an integer, the equally-optimal solutions will have difference-vectors with corrective factors of any number between the two medians.
So I guess this wouldn't be complete without a final example.
input: 2 3 4 10 14 14 15 100
difference vector: 2 3 4 5 6 7 8 9-2 3 4 10 14 14 15 100 = 0 0 0 -5 -8 -7 -7 -91
note that the medians of the difference-vector are not in the middle anymore, we need to perform an O(N) median-finding algorithm to extract them...
medians of difference-vector are -5 and -7
let us take -5 to be our correction factor (any number between the medians, such as -6 or -7, would also be a valid choice)
thus our new goal is 2 3 4 5 6 7 8 9+5=7 8 9 10 11 12 13 14, and the new differences are 5 5 5 0 -3 -2 -2 -86*
this means we will need to do 5+5+5+0+3+2+2+86=108 steps
*(we obtain this by repeating step 2 with our new target, or by adding 5 to each number of the previous difference... but since you only care about the sum, we'd just add 8*5 (vector length times correct factor) to the previously calculated sum)
Alternatively, we could have also taken -6 or -7 to be our correction factor. Let's say we took -7...
then the new goal would have been 2 3 4 5 6 7 8 9+7=9 10 11 12 13 14 15 16, and the new differences would have been 7 7 7 2 1 0 0 -84
this would have meant we'd need to do 7+7+7+2+1+0+0+84=108 steps, the same as above
If you simulate this yourself, can see the number of steps becomes >108 as we take offsets further away from the range [-5,-7].
Pseudocode:
def minSteps(array A of size N):
A' = [0,1,...,N-1]
diffs = A'-A
medianOfDiffs = leftMedian(diffs)
return sum(abs(diffs-medianOfDiffs))
Python:
leftMedian = lambda x:sorted(x)[len(x)//2]
def minSteps(array):
target = range(len(array))
diffs = [t-a for t,a in zip(target,array)]
medianOfDiffs = leftMedian(diffs)
return sum(abs(d-medianOfDiffs) for d in diffs)
edit:
It turns out that for arrays of distinct integers, this is equivalent to a simpler solution: picking one of the (up to 2) medians, assuming it doesn't move, and moving other numbers accordingly. This simpler method often gives incorrect answers if you have any duplicates, but the OP didn't ask that, so that would be a simpler and more elegant solution. Additionally we can use the proof I've given in this solution to justify the "assume the median doesn't move" solution as follows: the corrective factor will always be in the center of the array (i.e. the median of the differences will be from the median of the numbers). Thus any restriction which also guarantees this can be used to create variations of this brainteaser.
Get one of the medians of all the numbers. As the numbers are already sorted, this shouldn't be a big deal. Assume that median does not move. Then compute the total cost of moving all the numbers accordingly. This should give the answer.
community edit:
def minSteps(a):
"""INPUT: list of sorted unique integers"""
oneMedian = a[floor(n/2)]
aTarget = [oneMedian + (i-floor(n/2)) for i in range(len(a))]
# aTargets looks roughly like [m-n/2?, ..., m-1, m, m+1, ..., m+n/2]
return sum(abs(aTarget[i]-a[i]) for i in range(len(a)))
This is probably not an ideal solution, but a first idea.
Given a sorted sequence [x1, x2, …, xn]:
Write a function that returns the differences of an element to the previous and to the next element, i.e. (xn – xn–1, xn+1 – xn).
If the difference to the previous element is > 1, you would have to increase all previous elements by xn – xn–1 – 1. That is, the number of necessary steps would increase by the number of previous elements × (xn – xn–1 – 1). Let's call this number a.
If the difference to the next element is >1, you would have to decrease all subsequent elements by xn+1 – xn – 1. That is, the number of necessary steps would increase by the number of subsequent elements × (xn+1 – xn – 1). Let's call this number b.
If a < b, then increase all previous elements until they are contiguous to the current element. If a > b, then decrease all subsequent elements until they are contiguous to the current element. If a = b, it doesn't matter which of these two actions is chosen.
Add up the number of steps taken in the previous step (by increasing the total number of necessary steps by either a or b), and repeat until all elements are contiguous.
First of all, imagine that we pick an arbitrary target of contiguous increasing values and then calculate the cost (number of steps required) for modifying the array the array to match.
Original: 3 5 7 8 10 16
Target: 4 5 6 7 8 9
Difference: +1 0 -1 -1 -2 -7 -> Cost = 12
Sign: + 0 - - - -
Because the input array is already ordered and distinct, it is strictly increasing. Because of this, it can be shown that the differences will always be non-increasing.
If we change the target by increasing it by 1, the cost will change. Each position in which the difference is currently positive or zero will incur an increase in cost by 1. Each position in which the difference is currently negative will yield a decrease in cost by 1:
Original: 3 5 7 8 10 16
New target: 5 6 7 8 9 10
New Difference: +2 +1 0 0 -1 -6 -> Cost = 10 (decrease by 2)
Conversely, if we decrease the target by 1, each position in which the difference is currently positive will yield a decrease in cost by 1, while each position in which the difference is zero or negative will incur an increase in cost by 1:
Original: 3 5 7 8 10 16
New target: 3 4 5 6 7 8
New Difference: 0 -1 -2 -2 -3 -8 -> Cost = 16 (increase by 4)
In order to find the optimal values for the target array, we must find a target such that any change (increment or decrement) will not decrease the cost. Note that an increment of the target can only decrease the cost when there are more positions with negative difference than there are with zero or positive difference. A decrement can only decrease the cost when there are more positions with a positive difference than with a zero or negative difference.
Here are some example distributions of difference signs. Remember that the differences array is non-increasing, so positives always have to be first and negatives last:
C C
+ + + - - - optimal
+ + 0 - - - optimal
0 0 0 - - - optimal
+ 0 - - - - can increment (negatives exceed positives & zeroes)
+ + + 0 0 0 optimal
+ + + + - - can decrement (positives exceed negatives & zeroes)
+ + 0 0 - - optimal
+ 0 0 0 0 0 optimal
C C
Observe that if one of the central elements (marked C) is zero, the target must be optimal. In such a circumstance, at best any increment or decrement will not change the cost, but it may increase it. This result is important, because it gives us a trivial solution. We pick a target such that a[n/2] remains unchanged. There may be other possible targets that yield the same cost, but there are definitely none that are better. Here's the original code modified to calculate this cost:
//n is the number of elements in array a
int targetValue;
int cost = 0;
int middle = n / 2;
int startValue = a[middle] - middle;
for (i = 0; i < n; i++)
{
targetValue = startValue + i;
cost += abs(targetValue - a[i]);
}
printf("%d\n",cost);
You can not do it by iterating once on the array, that's for sure.
You need first to check the difference between each two numbers, for example:
2,7,8,9 can be 2,3,4,5 with 18 steps or 6,7,8,9 with 4 steps.
Create a new array with the difference like so: for 2,7,8,9 it wiil be 4,1,1. Now you can decide whether to increase or decrease the first number.
Lets assume that the contiguous array looks something like this -
c c+1 c+2 c+3 .. and so on
Now lets take an example -
5 7 8 10
The contiguous array in this case will be -
c c+1 c+2 c+3
In order to get the minimum steps, the sum of the modulus of the difference of the integers(before and after) w.r.t the ith index should be the minimum. In which case,
(c-5)^2 + (c-6)^2 + (c-6)^2 + (c-7)^2 should be minimum
Let f(c) = (c-5)^2 + (c-6)^2 + (c-6)^2 + (c-7)^2
= 4c^2 - 48c + 146
Applying differential calculus to get the minima,
f'(c) = 8c - 48 = 0
=> c = 6
So our contiguous array is 6 7 8 9 and the minimum cost here is 2.
To sum it up, just generate f(c), get the first differential and find out c.
This should take O(n).
Brute force approach O(N*M)
If one draws a line through each point in the array a then y0 is a value where each line starts at index 0. Then the answer is the minimum among number of steps reqired to get from a to every line that starts at y0, in Python:
y0s = set((y - i) for i, y in enumerate(a))
nsteps = min(sum(abs(y-(y0+i)) for i, y in enumerate(a))
for y0 in xrange(min(y0s), max(y0s)+1)))
Input
2,4,5,6
2,4,5,8
Output
1
3

Resources